Is my Venn diagram for probability of event B correct?

  • Thread starter Thread starter tzx9633
  • Start date Start date
  • Tags Tags
    Confusion Diagram
Click For Summary
The discussion centers on the probability of event B given mutually exclusive and exhaustive events A1, A2, ..., An. The correct formula for calculating P(B) is presented as a sum of the products of the probabilities of each Ai and the conditional probabilities of B given Ai. It is emphasized that this formula holds true only if event B is a subset of the union of all Ai. The accuracy of the Venn diagram representation is questioned, specifically regarding whether B is depicted as a subset of the union of the Ai. The conclusion is that the diagram's correctness hinges on this subset relationship.
tzx9633

Homework Statement


It's given that the total probability of event b =P(B)

If A1 , A2 ... An are mutually exclusive and exhaustive events , then probability of B =



Homework Equations

The Attempt at a Solution


P(B) = P(A1) x P(B | A1) + P(A2) x P(B | A2) + P(A3) x P(B | A3) + ... P(An) x P(B | An)

IS it correct if i represent them in this Venn's diagram ?
 

Attachments

  • 691.png
    691.png
    4.1 KB · Views: 545
Physics news on Phys.org
tzx9633 said:
IS it correct if i represent them in this Venn's diagram ?
That depends. Is the region not included in any of the blue circles one of the Ai?
 
  • Like
Likes Ray Vickson
tzx9633 said:

Homework Statement


It's given that the total probability of event b =P(B)

If A1 , A2 ... An are mutually exclusive and exhaustive events , then probability of B =



Homework Equations

The Attempt at a Solution


P(B) = P(A1) x P(B | A1) + P(A2) x P(B | A2) + P(A3) x P(B | A3) + ... P(An) x P(B | An)

IS it correct if i represent them in this Venn's diagram ?

The statement ##P(B) = \sum_{i=1}^n P(A_i) P(B|A_i)## is correct ONLY if ##B \subset \cup_{i=1}^n A_i##. In your case you said the ##\{ A_i \}## are mutually exclusive and exhaustive (meaning that ##\cup_{i=1}^n A_i## is the whole sample space), so the equation is correct. I guess you are asking if your Venn diagram is also correct. Well, do you have ##B \subset \cup_{i=1}^n A_i## in your diagram?
 
Question: A clock's minute hand has length 4 and its hour hand has length 3. What is the distance between the tips at the moment when it is increasing most rapidly?(Putnam Exam Question) Answer: Making assumption that both the hands moves at constant angular velocities, the answer is ## \sqrt{7} .## But don't you think this assumption is somewhat doubtful and wrong?

Similar threads

  • · Replies 9 ·
Replies
9
Views
2K
  • · Replies 6 ·
Replies
6
Views
2K
  • · Replies 2 ·
Replies
2
Views
3K
  • · Replies 8 ·
Replies
8
Views
5K
  • · Replies 2 ·
Replies
2
Views
2K
  • · Replies 3 ·
Replies
3
Views
2K
Replies
3
Views
3K
  • · Replies 3 ·
Replies
3
Views
2K
  • · Replies 3 ·
Replies
3
Views
8K
  • · Replies 1 ·
Replies
1
Views
1K